Difference between revisions of "2002 AIME I Problems/Problem 4"

m (See also)
Line 3: Line 3:
  
 
== Solution ==
 
== Solution ==
{{solution}}
+
<math>\dfrac{1}{k^2+k}=\dfrac{1}{k(k+1)}=\dfrac{1}{k}-\dfrac{1}{k+1}</math>. Thus,
 +
 
 +
<math>a_m+a_{m+1}+a_{m+2}+\cdots +a_{n-1}=\dfrac{1}{m}-\dfrac{1}{m+1}+\dfrac{1}{m+1}-\dfrac{1}{m+2}+\cdots +\dfrac{1}{n-1}-\dfrac{1}{n-2}=\dfrac{1}{m}-\dfrac{1}{n-2}</math>
 +
 
 +
Which is
 +
 
 +
<math>\dfrac{n-2-m}{m(n-2)}=\dfrac{1}{29}</math>
 +
 
 +
We cross-multiply to get
 +
 
 +
<math>mn-2m=29n-58-29m\Rightarrow mn+27m-29n+58=0\Rightarrow (m-29)(n+27)=-725</math>
 +
 
 +
Thus <math>m</math> is an integer less than 29. We try <math>m=4</math> to get <math>n=2</math>. <math>m=24, n=118</math>. <math>m+n=\boxed{142}</math>
  
 
== See also ==
 
== See also ==
 
{{AIME box|year=2002|n=I|num-b=3|num-a=5}}
 
{{AIME box|year=2002|n=I|num-b=3|num-a=5}}

Revision as of 09:07, 5 May 2008

Problem

Consider the sequence defined by $a_k =\dfrac{1}{k^2+k}$ for $k\geq 1$. Given that $a_m+a_{m+1}+\cdots+a_{n-1}=\dfrac{1}{29}$, for positive integers $m$ and $n$ with $m<n$, find $m+n$.

Solution

$\dfrac{1}{k^2+k}=\dfrac{1}{k(k+1)}=\dfrac{1}{k}-\dfrac{1}{k+1}$. Thus,

$a_m+a_{m+1}+a_{m+2}+\cdots +a_{n-1}=\dfrac{1}{m}-\dfrac{1}{m+1}+\dfrac{1}{m+1}-\dfrac{1}{m+2}+\cdots +\dfrac{1}{n-1}-\dfrac{1}{n-2}=\dfrac{1}{m}-\dfrac{1}{n-2}$

Which is

$\dfrac{n-2-m}{m(n-2)}=\dfrac{1}{29}$

We cross-multiply to get

$mn-2m=29n-58-29m\Rightarrow mn+27m-29n+58=0\Rightarrow (m-29)(n+27)=-725$

Thus $m$ is an integer less than 29. We try $m=4$ to get $n=2$. $m=24, n=118$. $m+n=\boxed{142}$

See also

2002 AIME I (ProblemsAnswer KeyResources)
Preceded by
Problem 3
Followed by
Problem 5
1 2 3 4 5 6 7 8 9 10 11 12 13 14 15
All AIME Problems and Solutions